Is the Euclidean Topology of ℝn First Countable?

  • Thread starter Thread starter Hodgey8806
  • Start date Start date
  • Tags Tags
    Euclidean Topology
Click For Summary
The discussion confirms that the Euclidean topology of ℝn is first countable, as a countable neighborhood basis can be constructed using open balls Br(p) with rational radii. It is established that for any neighborhood U of a point p, there exists a rational radius r such that Br(p) is contained in U. A nested neighborhood basis at the point 5 can be defined using intervals B(1/2)i(5) for natural numbers i, ensuring that each subsequent interval is contained within the previous one. Participants clarify details regarding the necessity for r to be both rational and positive, and they acknowledge the role of the density of rationals in this context. Overall, the proof is validated with minor corrections noted.
Hodgey8806
Messages
140
Reaction score
3

Homework Statement


Let X:=ℝn with the Euclidean Topology. Is X first countable? Find a nested neighborhood basis for X at 5.


Homework Equations


If X is a topological space and p\inX, a collection Bp of neighborhoods of p is called a neighborhood basis for X at p if every neighborhood of p contains some B\inBp.

We say X is first countable if there exists a countable neighborhood basis at each point.


The Attempt at a Solution


I say yes.
Let p\inX, the set of open balls Br(p) for r being rational forms a neighborhood basis at p. (That is, for all neighborhoods U of p, there is a Br(p)\subseteqU)
Since p was arbitrary and this Bp is countable (since rationals are countable), X is first countable.

As well, we can just let the nest interval be defined as: B(1/2)i(5) for i being a natural number. Thus, B(1/2)i+1(5)<B(1/2)i(5).

I am struggling a bit at this level of proof honestly, and I'm trying to stay afloat. Thank you!
 
Physics news on Phys.org


Your proof looks fine. Just a few nitpicking details: first, r needs to be rational AND positive. Second, although it's pretty obvious, you might say a few words about why you can find an r such that B_r(p) \subseteq U.

Your nested neighborhood basis at 5 is fine. Just one minor detail: instead of <, you want \subset.
 


Thank you!

Wouldn't it just be based on the density of the rationals?
Also, thank you for catching my typo :)
 


Thanks again! This site is amazing!
 
Question: A clock's minute hand has length 4 and its hour hand has length 3. What is the distance between the tips at the moment when it is increasing most rapidly?(Putnam Exam Question) Answer: Making assumption that both the hands moves at constant angular velocities, the answer is ## \sqrt{7} .## But don't you think this assumption is somewhat doubtful and wrong?

Similar threads

Replies
1
Views
2K
  • · Replies 8 ·
Replies
8
Views
2K
  • · Replies 4 ·
Replies
4
Views
3K
  • · Replies 7 ·
Replies
7
Views
1K
  • · Replies 3 ·
Replies
3
Views
3K
  • · Replies 1 ·
Replies
1
Views
2K
  • · Replies 1 ·
Replies
1
Views
1K
  • · Replies 20 ·
Replies
20
Views
3K
  • · Replies 15 ·
Replies
15
Views
2K
  • · Replies 17 ·
Replies
17
Views
3K